How to proceed with this integral?

The name of the pictureThe name of the pictureThe name of the pictureClash Royale CLAN TAG#URR8PPP











up vote
3
down vote

favorite
1













Let $ G:= left (x,y) in mathbbR^2 : 0 < y,: x^2 + fracy^29 <1: ,: x^2+y^2 > 1 right $.



I want to calculate this integral:
$ displaystyleint_G x^2,dxdy $.




I want to try with polar coordinates:
so I set $ (x,y) = (rcosphi,rsinphi)$, but
I am not sure how to get the right boundaries for $phi $. Isn't it $ x^2 +y^2 = r^2 $ ?



Any help is very appreciated !










share|cite|improve this question



























    up vote
    3
    down vote

    favorite
    1













    Let $ G:= left (x,y) in mathbbR^2 : 0 < y,: x^2 + fracy^29 <1: ,: x^2+y^2 > 1 right $.



    I want to calculate this integral:
    $ displaystyleint_G x^2,dxdy $.




    I want to try with polar coordinates:
    so I set $ (x,y) = (rcosphi,rsinphi)$, but
    I am not sure how to get the right boundaries for $phi $. Isn't it $ x^2 +y^2 = r^2 $ ?



    Any help is very appreciated !










    share|cite|improve this question

























      up vote
      3
      down vote

      favorite
      1









      up vote
      3
      down vote

      favorite
      1






      1






      Let $ G:= left (x,y) in mathbbR^2 : 0 < y,: x^2 + fracy^29 <1: ,: x^2+y^2 > 1 right $.



      I want to calculate this integral:
      $ displaystyleint_G x^2,dxdy $.




      I want to try with polar coordinates:
      so I set $ (x,y) = (rcosphi,rsinphi)$, but
      I am not sure how to get the right boundaries for $phi $. Isn't it $ x^2 +y^2 = r^2 $ ?



      Any help is very appreciated !










      share|cite|improve this question
















      Let $ G:= left (x,y) in mathbbR^2 : 0 < y,: x^2 + fracy^29 <1: ,: x^2+y^2 > 1 right $.



      I want to calculate this integral:
      $ displaystyleint_G x^2,dxdy $.




      I want to try with polar coordinates:
      so I set $ (x,y) = (rcosphi,rsinphi)$, but
      I am not sure how to get the right boundaries for $phi $. Isn't it $ x^2 +y^2 = r^2 $ ?



      Any help is very appreciated !







      calculus integration definite-integrals multiple-integral






      share|cite|improve this question















      share|cite|improve this question













      share|cite|improve this question




      share|cite|improve this question








      edited Nov 22 at 8:28









      Robert Z

      90.8k1057128




      90.8k1057128










      asked Nov 22 at 8:10









      wondering1123

      1249




      1249




















          3 Answers
          3






          active

          oldest

          votes

















          up vote
          9
          down vote













          Even though I believe that the other answers are the best way to go, I think that you can use polar coordinates if you wish.



          Since $y>0$, we are integrating over the first two quadrants, so that $0 leq phi leq pi$.
          Since $x^2+y^2> 1$, we have $r > 1$, and since $x^2+y^2/9 < 1$, we see that
          $$ r^2cos^2theta + fracr^2sin^2theta9 < 1 quad Rightarrow quad r < frac3sqrt8cos^2theta+1. $$



          Thus
          beginalign
          int_0^pi int_1^frac3sqrt8cos^2theta+1 r^2cos^2theta cdot r ,mathrm dr , mathrm dtheta &= frac814 int_0^pi fraccos^2theta(8cos^2theta+1)^2 , mathrm dtheta - frac14 int_0^pi cos^2theta , mathrm dtheta = fracpi4.
          endalign






          share|cite|improve this answer





























            up vote
            5
            down vote













            It is not a good idea to use polar coordinates. The integral can be written as $int_-1^1int_sqrt1-x^2 ^3sqrt1-x^2x^2, dy, dx=int_-1^12sqrt 1-x^2x^2, dx$. To evaluate this put $x=sin, theta$ and use the formulas $2sin, theta cos, theta =sin, 2theta$, $2sin^2, 2theta =1-cos (4theta)$.






            share|cite|improve this answer



























              up vote
              5
              down vote













              Why in polar coordinates? Maybe it is easier by cartesian coordinates. Note that in $G$, $-1leq xleq 1$ and
              $$sqrt1-x^2<y<3sqrt1-x^2.$$
              Therefore
              $$int_G x^2,dxdy=int_x=-1^1x^2left(3sqrt1-x^2-sqrt1-x^2right)dx$$
              Can you take it from here?






              share|cite|improve this answer






















                Your Answer





                StackExchange.ifUsing("editor", function ()
                return StackExchange.using("mathjaxEditing", function ()
                StackExchange.MarkdownEditor.creationCallbacks.add(function (editor, postfix)
                StackExchange.mathjaxEditing.prepareWmdForMathJax(editor, postfix, [["$", "$"], ["\\(","\\)"]]);
                );
                );
                , "mathjax-editing");

                StackExchange.ready(function()
                var channelOptions =
                tags: "".split(" "),
                id: "69"
                ;
                initTagRenderer("".split(" "), "".split(" "), channelOptions);

                StackExchange.using("externalEditor", function()
                // Have to fire editor after snippets, if snippets enabled
                if (StackExchange.settings.snippets.snippetsEnabled)
                StackExchange.using("snippets", function()
                createEditor();
                );

                else
                createEditor();

                );

                function createEditor()
                StackExchange.prepareEditor(
                heartbeatType: 'answer',
                convertImagesToLinks: true,
                noModals: true,
                showLowRepImageUploadWarning: true,
                reputationToPostImages: 10,
                bindNavPrevention: true,
                postfix: "",
                imageUploader:
                brandingHtml: "Powered by u003ca class="icon-imgur-white" href="https://imgur.com/"u003eu003c/au003e",
                contentPolicyHtml: "User contributions licensed under u003ca href="https://creativecommons.org/licenses/by-sa/3.0/"u003ecc by-sa 3.0 with attribution requiredu003c/au003e u003ca href="https://stackoverflow.com/legal/content-policy"u003e(content policy)u003c/au003e",
                allowUrls: true
                ,
                noCode: true, onDemand: true,
                discardSelector: ".discard-answer"
                ,immediatelyShowMarkdownHelp:true
                );



                );













                 

                draft saved


                draft discarded


















                StackExchange.ready(
                function ()
                StackExchange.openid.initPostLogin('.new-post-login', 'https%3a%2f%2fmath.stackexchange.com%2fquestions%2f3008866%2fhow-to-proceed-with-this-integral%23new-answer', 'question_page');

                );

                Post as a guest















                Required, but never shown

























                3 Answers
                3






                active

                oldest

                votes








                3 Answers
                3






                active

                oldest

                votes









                active

                oldest

                votes






                active

                oldest

                votes








                up vote
                9
                down vote













                Even though I believe that the other answers are the best way to go, I think that you can use polar coordinates if you wish.



                Since $y>0$, we are integrating over the first two quadrants, so that $0 leq phi leq pi$.
                Since $x^2+y^2> 1$, we have $r > 1$, and since $x^2+y^2/9 < 1$, we see that
                $$ r^2cos^2theta + fracr^2sin^2theta9 < 1 quad Rightarrow quad r < frac3sqrt8cos^2theta+1. $$



                Thus
                beginalign
                int_0^pi int_1^frac3sqrt8cos^2theta+1 r^2cos^2theta cdot r ,mathrm dr , mathrm dtheta &= frac814 int_0^pi fraccos^2theta(8cos^2theta+1)^2 , mathrm dtheta - frac14 int_0^pi cos^2theta , mathrm dtheta = fracpi4.
                endalign






                share|cite|improve this answer


























                  up vote
                  9
                  down vote













                  Even though I believe that the other answers are the best way to go, I think that you can use polar coordinates if you wish.



                  Since $y>0$, we are integrating over the first two quadrants, so that $0 leq phi leq pi$.
                  Since $x^2+y^2> 1$, we have $r > 1$, and since $x^2+y^2/9 < 1$, we see that
                  $$ r^2cos^2theta + fracr^2sin^2theta9 < 1 quad Rightarrow quad r < frac3sqrt8cos^2theta+1. $$



                  Thus
                  beginalign
                  int_0^pi int_1^frac3sqrt8cos^2theta+1 r^2cos^2theta cdot r ,mathrm dr , mathrm dtheta &= frac814 int_0^pi fraccos^2theta(8cos^2theta+1)^2 , mathrm dtheta - frac14 int_0^pi cos^2theta , mathrm dtheta = fracpi4.
                  endalign






                  share|cite|improve this answer
























                    up vote
                    9
                    down vote










                    up vote
                    9
                    down vote









                    Even though I believe that the other answers are the best way to go, I think that you can use polar coordinates if you wish.



                    Since $y>0$, we are integrating over the first two quadrants, so that $0 leq phi leq pi$.
                    Since $x^2+y^2> 1$, we have $r > 1$, and since $x^2+y^2/9 < 1$, we see that
                    $$ r^2cos^2theta + fracr^2sin^2theta9 < 1 quad Rightarrow quad r < frac3sqrt8cos^2theta+1. $$



                    Thus
                    beginalign
                    int_0^pi int_1^frac3sqrt8cos^2theta+1 r^2cos^2theta cdot r ,mathrm dr , mathrm dtheta &= frac814 int_0^pi fraccos^2theta(8cos^2theta+1)^2 , mathrm dtheta - frac14 int_0^pi cos^2theta , mathrm dtheta = fracpi4.
                    endalign






                    share|cite|improve this answer














                    Even though I believe that the other answers are the best way to go, I think that you can use polar coordinates if you wish.



                    Since $y>0$, we are integrating over the first two quadrants, so that $0 leq phi leq pi$.
                    Since $x^2+y^2> 1$, we have $r > 1$, and since $x^2+y^2/9 < 1$, we see that
                    $$ r^2cos^2theta + fracr^2sin^2theta9 < 1 quad Rightarrow quad r < frac3sqrt8cos^2theta+1. $$



                    Thus
                    beginalign
                    int_0^pi int_1^frac3sqrt8cos^2theta+1 r^2cos^2theta cdot r ,mathrm dr , mathrm dtheta &= frac814 int_0^pi fraccos^2theta(8cos^2theta+1)^2 , mathrm dtheta - frac14 int_0^pi cos^2theta , mathrm dtheta = fracpi4.
                    endalign







                    share|cite|improve this answer














                    share|cite|improve this answer



                    share|cite|improve this answer








                    edited Nov 22 at 13:05

























                    answered Nov 22 at 8:37









                    MisterRiemann

                    5,0581623




                    5,0581623




















                        up vote
                        5
                        down vote













                        It is not a good idea to use polar coordinates. The integral can be written as $int_-1^1int_sqrt1-x^2 ^3sqrt1-x^2x^2, dy, dx=int_-1^12sqrt 1-x^2x^2, dx$. To evaluate this put $x=sin, theta$ and use the formulas $2sin, theta cos, theta =sin, 2theta$, $2sin^2, 2theta =1-cos (4theta)$.






                        share|cite|improve this answer
























                          up vote
                          5
                          down vote













                          It is not a good idea to use polar coordinates. The integral can be written as $int_-1^1int_sqrt1-x^2 ^3sqrt1-x^2x^2, dy, dx=int_-1^12sqrt 1-x^2x^2, dx$. To evaluate this put $x=sin, theta$ and use the formulas $2sin, theta cos, theta =sin, 2theta$, $2sin^2, 2theta =1-cos (4theta)$.






                          share|cite|improve this answer






















                            up vote
                            5
                            down vote










                            up vote
                            5
                            down vote









                            It is not a good idea to use polar coordinates. The integral can be written as $int_-1^1int_sqrt1-x^2 ^3sqrt1-x^2x^2, dy, dx=int_-1^12sqrt 1-x^2x^2, dx$. To evaluate this put $x=sin, theta$ and use the formulas $2sin, theta cos, theta =sin, 2theta$, $2sin^2, 2theta =1-cos (4theta)$.






                            share|cite|improve this answer












                            It is not a good idea to use polar coordinates. The integral can be written as $int_-1^1int_sqrt1-x^2 ^3sqrt1-x^2x^2, dy, dx=int_-1^12sqrt 1-x^2x^2, dx$. To evaluate this put $x=sin, theta$ and use the formulas $2sin, theta cos, theta =sin, 2theta$, $2sin^2, 2theta =1-cos (4theta)$.







                            share|cite|improve this answer












                            share|cite|improve this answer



                            share|cite|improve this answer










                            answered Nov 22 at 8:20









                            Kavi Rama Murthy

                            42.8k31751




                            42.8k31751




















                                up vote
                                5
                                down vote













                                Why in polar coordinates? Maybe it is easier by cartesian coordinates. Note that in $G$, $-1leq xleq 1$ and
                                $$sqrt1-x^2<y<3sqrt1-x^2.$$
                                Therefore
                                $$int_G x^2,dxdy=int_x=-1^1x^2left(3sqrt1-x^2-sqrt1-x^2right)dx$$
                                Can you take it from here?






                                share|cite|improve this answer


























                                  up vote
                                  5
                                  down vote













                                  Why in polar coordinates? Maybe it is easier by cartesian coordinates. Note that in $G$, $-1leq xleq 1$ and
                                  $$sqrt1-x^2<y<3sqrt1-x^2.$$
                                  Therefore
                                  $$int_G x^2,dxdy=int_x=-1^1x^2left(3sqrt1-x^2-sqrt1-x^2right)dx$$
                                  Can you take it from here?






                                  share|cite|improve this answer
























                                    up vote
                                    5
                                    down vote










                                    up vote
                                    5
                                    down vote









                                    Why in polar coordinates? Maybe it is easier by cartesian coordinates. Note that in $G$, $-1leq xleq 1$ and
                                    $$sqrt1-x^2<y<3sqrt1-x^2.$$
                                    Therefore
                                    $$int_G x^2,dxdy=int_x=-1^1x^2left(3sqrt1-x^2-sqrt1-x^2right)dx$$
                                    Can you take it from here?






                                    share|cite|improve this answer














                                    Why in polar coordinates? Maybe it is easier by cartesian coordinates. Note that in $G$, $-1leq xleq 1$ and
                                    $$sqrt1-x^2<y<3sqrt1-x^2.$$
                                    Therefore
                                    $$int_G x^2,dxdy=int_x=-1^1x^2left(3sqrt1-x^2-sqrt1-x^2right)dx$$
                                    Can you take it from here?







                                    share|cite|improve this answer














                                    share|cite|improve this answer



                                    share|cite|improve this answer








                                    edited Nov 22 at 8:33

























                                    answered Nov 22 at 8:20









                                    Robert Z

                                    90.8k1057128




                                    90.8k1057128



























                                         

                                        draft saved


                                        draft discarded















































                                         


                                        draft saved


                                        draft discarded














                                        StackExchange.ready(
                                        function ()
                                        StackExchange.openid.initPostLogin('.new-post-login', 'https%3a%2f%2fmath.stackexchange.com%2fquestions%2f3008866%2fhow-to-proceed-with-this-integral%23new-answer', 'question_page');

                                        );

                                        Post as a guest















                                        Required, but never shown





















































                                        Required, but never shown














                                        Required, but never shown












                                        Required, but never shown







                                        Required, but never shown

































                                        Required, but never shown














                                        Required, but never shown












                                        Required, but never shown







                                        Required, but never shown






                                        Popular posts from this blog

                                        How to check contact read email or not when send email to Individual?

                                        Bahrain

                                        Postfix configuration issue with fips on centos 7; mailgun relay